Difference between revisions of "2002 AMC 10P Problems"

(Problem 3)
(Problem 25)
 
(74 intermediate revisions by the same user not shown)
Line 35: Line 35:
  
 
== Problem 3 ==
 
== Problem 3 ==
Mary typed a six-digit number, but the two 1s she typed didn't show. What appeared was <math>2002.</math> How many different six-digit numbers could she have typed?
+
Mary typed a six-digit number, but the two <math>1</math>s she typed didn't show. What appeared was <math>2002.</math> How many different six-digit numbers could she have typed?
 +
 
 
<math>
 
<math>
 
\text{(A) }4
 
\text{(A) }4
Line 48: Line 49:
 
</math>
 
</math>
  
[[2002 AMC 12P Problems/Problem 3|Solution]]
+
[[2002 AMC 10P Problems/Problem 3|Solution]]
  
 
== Problem 4 ==
 
== Problem 4 ==
Let <math>a</math> and <math>b</math> be distinct real numbers for which
+
Which of the following numbers is a perfect square?
<cmath>\frac{a}{b} + \frac{a+10b}{b+10a} = 2.</cmath>
 
  
Find <math>\frac{a}{b}</math>
+
<math>\text{(A) }4^4 5^5 6^6 \qquad \text{(B) }4^4 5^6 6^5 \qquad \text{(C) }4^5 5^4 6^6 \qquad \text{(D) }4^6 5^4 6^5 \qquad \text{(E) }4^6 5^5 6^4</math>
  
<math>
+
[[2002 AMC 10P Problems/Problem 4|Solution]]
\text{(A) }0.4
 
\qquad
 
\text{(B) }0.5
 
\qquad
 
\text{(C) }0.6
 
\qquad
 
\text{(D) }0.7
 
\qquad
 
\text{(E) }0.8
 
</math>
 
 
 
[[2002 AMC 12P Problems/Problem 4|Solution]]
 
  
 
== Problem 5 ==
 
== Problem 5 ==
For how many positive integers <math>m</math> is
+
Let <math>(a_n)_{n \geq 1}</math> be a sequence such that <math>a_1 = 1</math> and <math>3a_{n+1} - 3a_n = 1</math> for all <math>n \geq 1.</math> Find <math>a_{2002}.</math>
<cmath>\frac{2002}{m^2 -2}</cmath>
 
 
 
a positive integer?
 
  
 
<math>
 
<math>
\text{(A) one}
+
\text{(A) }666
 
\qquad
 
\qquad
\text{(B) two}
+
\text{(B) }667
 
\qquad
 
\qquad
\text{(C) three}
+
\text{(C) }668
 
\qquad
 
\qquad
\text{(D) four}
+
\text{(D) }669
 
\qquad
 
\qquad
\text{(E) more than four}
+
\text{(E) }670
 
</math>
 
</math>
  
[[2002 AMC 12P Problems/Problem 5|Solution]]
+
[[2002 AMC 10P Problems/Problem 5|Solution]]
  
 
== Problem 6 ==
 
== Problem 6 ==
Participation in the local soccer league this year is <math>10\%</math> higher than last year. The number of males increased by <math>5\%</math> and the number of females increased by <math>20\%</math>. What fraction of the soccer league is now female?
+
The perimeter of a rectangle <math>100</math> and its diagonal has length <math>x.</math> What is the area of this rectangle?
 
 
 
<math>
 
<math>
\text{(A) }\frac{1}{3}
+
\text{(A) }625-x^2
 
\qquad
 
\qquad
\text{(B) }\frac{4}{11}
+
\text{(B) }625-\frac{x^2}{2}
 
\qquad
 
\qquad
\text{(C) }\frac{2}{5}
+
\text{(C) }1250-x^2
 
\qquad
 
\qquad
\text{(D) }\frac{4}{9}
+
\text{(D) }1250-\frac{x^2}{2}
 
\qquad
 
\qquad
\text{(E) }\frac{1}{2}
+
\text{(E) }2500-\frac{x^2}{2}
 
</math>
 
</math>
  
[[2002 AMC 12P Problems/Problem 6|Solution]]
+
[[2002 AMC 10P Problems/Problem 6|Solution]]
  
 
== Problem 7 ==
 
== Problem 7 ==
 
+
The dimensions of a rectangular box in inches are all positive integers and the volume of the box is <math>2002</math> in<math>^3</math>. Find the minimum possible sum of the three dimensions.
How many three-digit numbers have at least one <math>2</math> and at least one <math>3</math>?
 
  
 
<math>
 
<math>
\text{(A) }52
+
\text{(A) }36
 
\qquad
 
\qquad
\text{(B) }54
+
\text{(B) }38
 
\qquad
 
\qquad
\text{(C) }56
+
\text{(C) }42
 
\qquad
 
\qquad
\text{(D) }58
+
\text{(D) }44
 
\qquad
 
\qquad
\text{(E) }60
+
\text{(E) }92
 
</math>
 
</math>
  
[[2002 AMC 12P Problems/Problem 7|Solution]]
+
[[2002 AMC 10P Problems/Problem 7|Solution]]
  
 
== Problem 8 ==
 
== Problem 8 ==
  
Let <math>AB</math> be a segment of length <math>26</math>, and let points <math>C</math> and <math>D</math> be located on <math>AB</math> such that <math>AC=1</math> and <math>AD=8</math>. Let <math>E</math> and <math>F</math> be points on one of the semicircles with diameter <math>AB</math> for which <math>EC</math> and <math>FD</math> are perpendicular to <math>AB</math>. Find <math>EF.</math>
+
How many ordered triples of positive integers <math>(x,y,z)</math> satisfy <math>(x^y)^z=64?</math>
  
 
<math>
 
<math>
 
\text{(A) }5
 
\text{(A) }5
 
\qquad
 
\qquad
\text{(B) }5 \sqrt{2}
+
\text{(B) }6
 
\qquad
 
\qquad
 
\text{(C) }7
 
\text{(C) }7
 
\qquad
 
\qquad
\text{(D) }7 \sqrt{2}
+
\text{(D) }8
 
\qquad
 
\qquad
\text{(E) }12
+
\text{(E) }9
 
</math>
 
</math>
  
[[2002 AMC 12P Problems/Problem 8|Solution]]
+
[[2002 AMC 10P Problems/Problem 8|Solution]]
  
 
== Problem 9 ==
 
== Problem 9 ==
  
Two walls and the ceiling of a room meet at right angles at point <math>P.</math> A fly is in the air one meter from one wall, eight meters from the other wall, and nine meters from point <math>P</math>. How many meters is the fly from the ceiling?
+
The function <math>f</math> is given by the table
 +
 
 +
<cmath>
 +
\begin{tabular}{|c||c|c|c|c|c|}
 +
\hline
 +
x & 1 & 2 & 3 & 4 & 5 \\
 +
\hline
 +
f(x) & 4 & 1 & 3 & 5 & 2 \\
 +
\hline
 +
\end{tabular}
 +
</cmath>
 +
 
 +
If <math>u_0=4</math> and <math>u_{n+1} = f(u_n)</math> for <math>n \ge 0</math>, find <math>u_{2002}</math>
  
 
<math>
 
<math>
\text{(A) }\sqrt{13}
+
\text{(A) }1
 
\qquad
 
\qquad
\text{(B) }\sqrt{14}
+
\text{(B) }2
 
\qquad
 
\qquad
\text{(C) }\sqrt{15}
+
\text{(C) }3
 
\qquad
 
\qquad
 
\text{(D) }4
 
\text{(D) }4
 
\qquad
 
\qquad
\text{(E) }\sqrt{17}
+
\text{(E) }5
 
</math>
 
</math>
  
[[2002 AMC 12P Problems/Problem 9|Solution]]
+
[[2002 AMC 10P Problems/Problem 2|Solution]]
  
 
== Problem 10 ==
 
== Problem 10 ==
  
Let <math>f_n (x) = \text{sin}^n x + \text{cos}^n x.</math> For how many <math>x</math> in <math>[0,\pi]</math> is it true that
+
Let <math>a</math> and <math>b</math> be distinct real numbers for which
 +
<cmath>\frac{a}{b} + \frac{a+10b}{b+10a} = 2.</cmath>
  
<cmath>6f_{4}(x)-4f_{6}(x)=2f_{2}(x)?</cmath>
+
Find <math>\frac{a}{b}</math>
  
 
<math>
 
<math>
\text{(A) }2
+
\text{(A) }0.4
 
\qquad
 
\qquad
\text{(B) }4
+
\text{(B) }0.5
 
\qquad
 
\qquad
\text{(C) }6
+
\text{(C) }0.6
 
\qquad
 
\qquad
\text{(D) }8
+
\text{(D) }0.7
 
\qquad
 
\qquad
\text{(E) more than }8
+
\text{(E) }0.8
 
</math>
 
</math>
  
[[2002 AMC 12P Problems/Problem 10|Solution]]
+
[[2002 AMC 10P Problems/Problem 10|Solution]]
  
 
== Problem 11 ==
 
== Problem 11 ==
  
Let <math>t_n = \frac{n(n+1)}{2}</math> be the <math>n</math>th triangular number. Find
+
Let <math>P(x)=kx^3 + 2k^2x^2+k^3.</math> Find the sum of all real numbers <math>k</math> for which <math>x-2</math> is a factor of <math>P(x).</math>
 
 
<cmath>\frac{1}{t_1} + \frac{1}{t_2} + \frac{1}{t_3} + ... + \frac{1}{t_{2002}}</cmath>
 
  
 
<math>
 
<math>
\text{(A) }\frac {4003}{2003}
+
\text{(A) }-8
 
\qquad
 
\qquad
\text{(B) }\frac {2001}{1001}
+
\text{(B) }-4
 
\qquad
 
\qquad
\text{(C) }\frac {4004}{2003}
+
\text{(C) }0
 
\qquad
 
\qquad
\text{(D) }\frac {4001}{2001}
+
\text{(D) }5
 
\qquad
 
\qquad
\text{(E) }2
+
\text{(E) }8
 
</math>
 
</math>
  
[[2002 AMC 12P Problems/Problem 11|Solution]]
+
[[2002 AMC 10P Problems/Problem 11|Solution]]
  
 
== Problem 12 ==
 
== Problem 12 ==
  
For how many positive integers <math>n</math> is <math>n^3 - 8n^2 + 20n - 13</math> a prime number?
+
For <math>f_n(x)=x^n</math> and <math>a \neq 1</math> consider
 +
 
 +
<math>\text{I. } (f_{11}(a)f_{13}(a))^{14}</math>
 +
 
 +
<math>\text{II. } f_{11}(a)f_{13}(a)f_{14}(a)</math>
 +
 
 +
<math>\text{III. } (f_{11}(f_{13}(a)))^{14}</math>
 +
 
 +
<math>\text{IV. } f_{11}(f_{13}(f_{14}(a)))</math>
 +
 
 +
Which of these equal <math>f_{2002}(a)?</math>
  
 
<math>
 
<math>
\text{(A) one}
+
\text{(A) I and II only}
 
\qquad
 
\qquad
\text{(B) two}
+
\text{(B) II and III only}
 
\qquad
 
\qquad
\text{(C) three}
+
\text{(C) III and IV only}
 
\qquad
 
\qquad
\text{(D) four}
+
\text{(D) II, III, and IV only}
 
\qquad
 
\qquad
\text{(E) more than four}
+
\text{(E) all of them}
 
</math>
 
</math>
  
[[2002 AMC 12P Problems/Problem 12|Solution]]
+
[[2002 AMC 10P Problems/Problem 12|Solution]]
  
 
== Problem 13 ==
 
== Problem 13 ==
  
What is the maximum value of <math>n</math> for which there is a set of distinct positive integers <math>k_1, k_2, ... k_n</math> for which
+
Participation in the local soccer league this year is <math>10\%</math> higher than last year. The number of males increased by <math>5\%</math> and the number of females increased by <math>20\%</math>. What fraction of the soccer league is now female?
 
 
<cmath>k^2_1 + k^2_2 + ... + k^2_n = 2002?</cmath>
 
  
 
<math>
 
<math>
\text{(A) }14
+
\text{(A) }\frac{1}{3}
 
\qquad
 
\qquad
\text{(B) }15
+
\text{(B) }\frac{4}{11}
 
\qquad
 
\qquad
\text{(C) }16
+
\text{(C) }\frac{2}{5}
 
\qquad
 
\qquad
\text{(D) }17
+
\text{(D) }\frac{4}{9}
 
\qquad
 
\qquad
\text{(E) }18
+
\text{(E) }\frac{1}{2}
 
</math>
 
</math>
  
[[2002 AMC 12P Problems/Problem 13|Solution]]
+
[[2002 AMC 10P Problems/Problem 13|Solution]]
  
 
== Problem 14 ==
 
== Problem 14 ==
  
Find <math>i + 2i^2 +3i^3 + . . . + 2002i^{2002}.</math>
+
The vertex <math>E</math> of a square <math>EFGH</math> is at the center of square <math>ABCD.</math> The length of a side of <math>ABCD</math> is <math>1</math> and the length of a side of <math>EFGH</math> is <math>2.</math> Side <math>EF</math> intersects <math>CD</math> at <math>I</math> and <math>EH</math> intersects <math>AD</math> at <math>J.</math> If angle <math>EID=60^{\circ},</math> the area of quadrilateral <math>EIDJ</math> is
  
 
<math>
 
<math>
\text{(A) }-999 + 1002i
+
\text{(A) }\frac{1}{4}
 
\qquad
 
\qquad
\text{(B) }-1002 + 999i
+
\text{(B) }\frac{\sqrt{3}}{6}
 
\qquad
 
\qquad
\text{(C) }-1001 + 1000i
+
\text{(C) }\frac{1}{3}
 
\qquad
 
\qquad
\text{(D) }-1002 + 1001i
+
\text{(D) }\frac{\sqrt{2}}{4}
 
\qquad
 
\qquad
\text{(E) }i
+
\text{(E) }\frac{\sqrt{3}}{2}
 
</math>
 
</math>
  
[[2002 AMC 12P Problems/Problem 14|Solution]]
+
[[2002 AMC 10P Problems/Problem 14|Solution]]
  
 
== Problem 15 ==
 
== Problem 15 ==
There are <math>1001</math> red marbles and <math>1001</math> black marbles in a box. Let <math>P_s</math> be the probability that two marbles drawn at random from the box are the same color, and let <math>P_d</math> be the probability that they are different colors. Find <math>|P_s-P_d|.</math>
+
 
 +
What is the smallest integer <math>n</math> for which any subset of <math>\{ 1, 2, 3, \; \dots \; , 20 \}</math> of size <math>n</math> must contain two numbers that differ by 8?
  
 
<math>
 
<math>
\text{(A) }0
+
\text{(A) }2
 
\qquad
 
\qquad
\text{(B) }\frac{1}{2002}
+
\text{(B) }8
 
\qquad
 
\qquad
\text{(C) }\frac{1}{2001}
+
\text{(C) }12
 
\qquad
 
\qquad
\text{(D) }\frac {2}{2001}
+
\text{(D) }13
 
\qquad
 
\qquad
\text{(E) }\frac{1}{1000}
+
\text{(E) }15
 
</math>
 
</math>
  
[[2002 AMC 12P Problems/Problem 15|Solution]]
+
[[2002 AMC 10P Problems/Problem 15|Solution]]
  
 
== Problem 16 ==
 
== Problem 16 ==
  
The altitudes of a triangle are <math>12, 15,</math> and <math>20.</math> The largest angle in this triangle is
+
Two walls and the ceiling of a room meet at right angles at point <math>P.</math> A fly is in the air one meter from one wall, eight meters from the other wall, and nine meters from point <math>P</math>. How many meters is the fly from the ceiling?
  
 
<math>
 
<math>
\text{(A) }72^\circ
+
\text{(A) }\sqrt{13}
 
\qquad
 
\qquad
\text{(B) }75^\circ
+
\text{(B) }\sqrt{14}
 
\qquad
 
\qquad
\text{(C) }90^\circ
+
\text{(C) }\sqrt{15}
 
\qquad
 
\qquad
\text{(D) }108^\circ
+
\text{(D) }4
 
\qquad
 
\qquad
\text{(E) }120^\circ
+
\text{(E) }\sqrt{17}
 
</math>
 
</math>
  
[[2002 AMC 12P Problems/Problem 16|Solution]]
+
[[2002 AMC 10P Problems/Problem 16|Solution]]
  
 
== Problem 17 ==
 
== Problem 17 ==
  
Let <math>f(x) = \sqrt{\sin^4{x} + 4 \cos^2{x}} - \sqrt{\cos^4{x} + 4 \sin^2{x}}.</math> An equivalent form of <math>f(x)</math> is
+
There are <math>1001</math> red marbles and <math>1001</math> black marbles in a box. Let <math>P_s</math> be the probability that two marbles drawn at random from the box are the same color, and let <math>P_d</math> be the probability that they are different colors. Find <math>|P_s-P_d|.</math>
  
 
<math>
 
<math>
\text{(A) }1-\sqrt{2}\sin{x}
+
\text{(A) }0
 
\qquad
 
\qquad
\text{(B) }-1+\sqrt{2}\cos{x}
+
\text{(B) }\frac{1}{2002}
 
\qquad
 
\qquad
\text{(C) }\cos{\frac{x}{2}} - \sin{\frac{x}{2}}
+
\text{(C) }\frac{1}{2001}
 
\qquad
 
\qquad
\text{(D) }\cos{x} - \sin{x}
+
\text{(D) }\frac {2}{2001}
 
\qquad
 
\qquad
\text{(E) }\cos{2x}
+
\text{(E) }\frac{1}{1000}
 
</math>
 
</math>
  
[[2002 AMC 12P Problems/Problem 17|Solution]]
+
[[2002 AMC 10P Problems/Problem 17|Solution]]
  
 
== Problem 18 ==
 
== Problem 18 ==
 +
 +
For how many positive integers <math>n</math> is <math>n^3 - 8n^2 + 20n - 13</math> a prime number?
 +
 +
<math>
 +
\text{(A) one}
 +
\qquad
 +
\text{(B) two}
 +
\qquad
 +
\text{(C) three}
 +
\qquad
 +
\text{(D) four}
 +
\qquad
 +
\text{(E) more than four}
 +
</math>
 +
 +
[[2002 AMC 10P Problems/Problem 18|Solution]]
 +
 +
== Problem 19 ==
  
 
If <math>a,b,c</math> are real numbers such that <math>a^2 + 2b =7</math>, <math>b^2 + 4c= -7,</math> and <math>c^2 + 6a= -14</math>, find <math>a^2 + b^2 + c^2.</math>
 
If <math>a,b,c</math> are real numbers such that <math>a^2 + 2b =7</math>, <math>b^2 + 4c= -7,</math> and <math>c^2 + 6a= -14</math>, find <math>a^2 + b^2 + c^2.</math>
Line 326: Line 347:
 
</math>
 
</math>
  
[[2002 AMC 12P Problems/Problem 18|Solution]]
+
[[2002 AMC 10P Problems/Problem 19|Solution]]
  
== Problem 19 ==
+
== Problem 20 ==
  
In quadrilateral <math>ABCD</math>, <math>m\angle B = m \angle C = 120^{\circ}, AB=3, BC=4,</math> and <math>CD=5.</math> Find the area of <math>ABCD.</math>
+
How many three-digit numbers have at least one <math>2</math> and at least one <math>3</math>?
  
 
<math>
 
<math>
\text{(A) }15
+
\text{(A) }52
 
\qquad
 
\qquad
\text{(B) }9 \sqrt{3}
+
\text{(B) }54
 
\qquad
 
\qquad
\text{(C) }\frac{45 \sqrt{3}}{4}
+
\text{(C) }56
 
\qquad
 
\qquad
\text{(D) }\frac{47 \sqrt{3}}{4}
+
\text{(D) }58
 
\qquad
 
\qquad
\text{(E) }15 \sqrt{3}
+
\text{(E) }60
 
</math>
 
</math>
  
[[2002 AMC 12P Problems/Problem 19|Solution]]
+
[[2002 AMC 10P Problems/Problem 20|Solution]]
  
== Problem 20 ==
+
== Problem 21 ==
  
 
Let <math>f</math> be a real-valued function such that
 
Let <math>f</math> be a real-valued function such that
Line 366: Line 387:
 
</math>
 
</math>
  
[[2002 AMC 12P Problems/Problem 20|Solution]]
+
[[2002 AMC 10P Problems/Problem 21|Solution]]
  
== Problem 21 ==
+
== Problem 22 ==
  
Let <math>a</math> and <math>b</math> be real numbers greater than <math>1</math> for which there exists a positive real number <math>c,</math> different from <math>1</math>, such that
+
In how many zeroes does the number <math>\frac{2002!}{(1001!)^2}</math> end?
 
 
<cmath>2(\log_a{c} + \log_b{c}) = 9\log_{ab}{c}.</cmath>
 
 
 
Find the largest possible value of <math>\log_a b.</math>
 
  
 
<math>
 
<math>
\text{(A) }\sqrt{2}
+
\text{(A) }0
 
\qquad
 
\qquad
\text{(B) }\sqrt{3}
+
\text{(B) }1
 
\qquad
 
\qquad
 
\text{(C) }2
 
\text{(C) }2
 
\qquad
 
\qquad
\text{(D) }\sqrt{6}
+
\text{(D) }200
 
\qquad
 
\qquad
\text{(E) }3
+
\text{(E) }400
 
</math>
 
</math>
  
[[2002 AMC 12P Problems/Problem 21|Solution]]
+
[[2002 AMC 10P Problems/Problem 22|Solution]]
  
== Problem 22 ==
+
== Problem 23 ==
  
Under the new AMC <math>10, 12</math> scoring method, <math>6</math> poitns are given for each correct answer, <math>2.5</math> points are given for each unanswered question, and no points are given for an incorrect answer. Some of the possible scores between <math>0</math> and <math>150</math> can be obtained in only one way, for example, the only way to obtain a score of <math>146.5</math> is to have <math>24</math> correct answers and one unanswered question. Some scores can be obtained in exactly two ways, for example, a score of <math>104.5</math> can be obtained with <math>17</math> correct answers, <math>1</math> unanswered question, and <math>7</math> incorrect, and also with <math>12</math> correct answers and <math>13</math> unanswered questions. There are (three) scores that can be obtained in exactly three ways. What is their sum?
+
Let
 
+
<cmath>a=\frac{1^2}{1} + \frac{2^2}{3} + \frac{3^2}{5} +  \; \dots \; + \frac{1001^2}{2001}</cmath>
 
 
<math>
 
\text{(A) }175
 
\qquad
 
\text{(B) }179.5
 
\qquad
 
\text{(C) }182
 
\qquad
 
\text{(D) }188.5
 
\qquad
 
\text{(E) }201
 
</math>
 
  
[[2002 AMC 12P Problems/Problem 22|Solution]]
+
and
  
== Problem 23 ==
+
<cmath>a=\frac{1^2}{3} + \frac{2^2}{5} + \frac{3^2}{7} +  \; \dots \; + \frac{1001^2}{2003}.</cmath>
  
The equation <math>z(z+i)(z+3i)=2002i</math> has a zero of the form <math>a+bi</math>, where <math>a</math> and <math>b</math> are positive real numbers. Find <math>a.</math>
+
Find the integer closest to <math>a-b.</math>
  
 
<math>
 
<math>
\text{(A) }\sqrt{118}
+
\text{(A) }500
 
\qquad
 
\qquad
\text{(B) }\sqrt{210}
+
\text{(B) }501
 
\qquad
 
\qquad
\text{(C) }2 \sqrt{210}
+
\text{(C) }999
 
\qquad
 
\qquad
\text{(D) }\sqrt{2002}
+
\text{(D) }1000
 
\qquad
 
\qquad
\text{(E) }100 \sqrt{2}
+
\text{(E) }1001
 
</math>
 
</math>
  
[[2002 AMC 12P Problems/Problem 23|Solution]]
+
[[2002 AMC 10P Problems/Problem 23|Solution]]
  
 
== Problem 24 ==
 
== Problem 24 ==
  
Let <math>ABCD</math> be a regular tetrahedron and Let <math>E</math> be a point inside the face <math>ABC.</math> Denote by <math>s</math> the sum of the distances from <math>E</math> to the faces <math>DAB, DBC, DCA,</math> and by <math>S</math> the sum of the distances from <math>E</math> to the edges <math>AB, BC, CA.</math> Then <math>\frac{s}{S}</math> equals
+
What is the maximum value of <math>n</math> for which there is a set of distinct positive integers <math>k_1, k_2, ... k_n</math> for which
 +
 
 +
<cmath>k^2_1 + k^2_2 + ... + k^2_n = 2002?</cmath>
  
 
<math>
 
<math>
\text{(A) }\sqrt{2}
+
\text{(A) }14
 
\qquad
 
\qquad
\text{(B) }\frac{2 \sqrt{2}}{3}
+
\text{(B) }15
 
\qquad
 
\qquad
\text{(C) }\frac{\sqrt{6}}{2}
+
\text{(C) }16
 
\qquad
 
\qquad
\text{(D) }2
+
\text{(D) }17
 
\qquad
 
\qquad
\text{(E) }3
+
\text{(E) }18
 
</math>
 
</math>
  
[[2002 AMC 12P Problems/Problem 24|Solution]]
+
[[2002 AMC 10P Problems/Problem 24|Solution]]
  
 
== Problem 25 ==
 
== Problem 25 ==
  
Let <math>a</math> and <math>b</math> be real numbers such that <math>\sin{a} + \sin{b} = \frac{\sqrt{2}}{2}</math> and <math>\cos {a} + \cos {b} = \frac{\sqrt{6}}{2}.</math> Find <math>\sin{(a+b)}.</math>
+
Under the new AMC <math>10, 12</math> scoring method, <math>6</math> points are given for each correct answer, <math>2.5</math> points are given for each unanswered question, and no points are given for an incorrect answer. Some of the possible scores between <math>0</math> and <math>150</math> can be obtained in only one way, for example, the only way to obtain a score of <math>146.5</math> is to have <math>24</math> correct answers and one unanswered question. Some scores can be obtained in exactly two ways, for example, a score of <math>104.5</math> can be obtained with <math>17</math> correct answers, <math>1</math> unanswered question, and <math>7</math> incorrect, and also with <math>12</math> correct answers and <math>13</math> unanswered questions. There are (three) scores that can be obtained in exactly three ways. What is their sum?
 +
 
  
 
<math>
 
<math>
\text{(A) }\frac{1}{2}
+
\text{(A) }175
 
\qquad
 
\qquad
\text{(B) }\frac{\sqrt{2}}{2}
+
\text{(B) }179.5
 
\qquad
 
\qquad
\text{(C) }\frac{\sqrt{3}}{2}
+
\text{(C) }182
 
\qquad
 
\qquad
\text{(D) }\frac{\sqrt{6}}{2}
+
\text{(D) }188.5
 
\qquad
 
\qquad
\text{(E) }1
+
\text{(E) }201
 
</math>
 
</math>
  
[[2002 AMC 12P Problems/Problem 25|Solution]]
+
[[2002 AMC 10P Problems/Problem 25|Solution]]
  
 
== See also ==
 
== See also ==
  
{{AMC12 box|year=2001|before=[[2000 AMC 12 Problems]]|after=[[2002 AMC 12A Problems]]}}
+
{{AMC10 box|year=2002|ab=P|before=[[2001 AMC 10 Problems]]|after=[[2002 AMC 10A Problems]]}}
  
* [[AMC 12]]
+
* [[AMC 10]]
* [[AMC 12 Problems and Solutions]]
+
* [[AMC 10 Problems and Solutions]]
* [[2002 AMC 12]]
+
* [[2002 AMC 10P]]
 
* [[Mathematics competition resources]]
 
* [[Mathematics competition resources]]
 
{{MAA Notice}}
 
{{MAA Notice}}

Latest revision as of 03:46, 14 July 2024

2002 AMC 10P (Answer Key)
Printable versions: WikiAoPS ResourcesPDF

Instructions

  1. This is a 25-question, multiple choice test. Each question is followed by answers marked A, B, C, D and E. Only one of these is correct.
  2. You will receive 6 points for each correct answer, 2.5 points for each problem left unanswered if the year is before 2006, 1.5 points for each problem left unanswered if the year is after 2006, and 0 points for each incorrect answer.
  3. No aids are permitted other than scratch paper, graph paper, ruler, compass, protractor and erasers (and calculators that are accepted for use on the SAT if before 2006. No problems on the test will require the use of a calculator).
  4. Figures are not necessarily drawn to scale.
  5. You will have 75 minutes working time to complete the test.
1 2 3 4 5 6 7 8 9 10 11 12 13 14 15 16 17 18 19 20 21 22 23 24 25

Problem 1

The ratio $\frac{(2^4)^8}{(4^8)^2}$ equals

$\text{(A) }\frac{1}{4} \qquad \text{(B) }\frac{1}{2} \qquad \text{(C) }1 \qquad \text{(D) }2 \qquad \text{(E) }8$

Solution

Problem 2

The sum of eleven consecutive integers is $2002.$ What is the smallest of these integers?

$\text{(A) }175 \qquad \text{(B) }177 \qquad \text{(C) }179 \qquad \text{(D) }180 \qquad \text{(E) }181$

Solution

Problem 3

Mary typed a six-digit number, but the two $1$s she typed didn't show. What appeared was $2002.$ How many different six-digit numbers could she have typed?

$\text{(A) }4 \qquad \text{(B) }8 \qquad \text{(C) }10 \qquad \text{(D) }15 \qquad \text{(E) }20$

Solution

Problem 4

Which of the following numbers is a perfect square?

$\text{(A) }4^4 5^5 6^6 \qquad \text{(B) }4^4 5^6 6^5 \qquad \text{(C) }4^5 5^4 6^6 \qquad \text{(D) }4^6 5^4 6^5 \qquad \text{(E) }4^6 5^5 6^4$

Solution

Problem 5

Let $(a_n)_{n \geq 1}$ be a sequence such that $a_1 = 1$ and $3a_{n+1} - 3a_n = 1$ for all $n \geq 1.$ Find $a_{2002}.$

$\text{(A) }666 \qquad \text{(B) }667 \qquad \text{(C) }668 \qquad \text{(D) }669 \qquad \text{(E) }670$

Solution

Problem 6

The perimeter of a rectangle $100$ and its diagonal has length $x.$ What is the area of this rectangle? $\text{(A) }625-x^2 \qquad \text{(B) }625-\frac{x^2}{2} \qquad \text{(C) }1250-x^2 \qquad \text{(D) }1250-\frac{x^2}{2} \qquad \text{(E) }2500-\frac{x^2}{2}$

Solution

Problem 7

The dimensions of a rectangular box in inches are all positive integers and the volume of the box is $2002$ in$^3$. Find the minimum possible sum of the three dimensions.

$\text{(A) }36 \qquad \text{(B) }38  \qquad \text{(C) }42 \qquad \text{(D) }44 \qquad \text{(E) }92$

Solution

Problem 8

How many ordered triples of positive integers $(x,y,z)$ satisfy $(x^y)^z=64?$

$\text{(A) }5 \qquad \text{(B) }6 \qquad \text{(C) }7 \qquad \text{(D) }8 \qquad \text{(E) }9$

Solution

Problem 9

The function $f$ is given by the table

\[\begin{tabular}{|c||c|c|c|c|c|}  \hline   x & 1 & 2 & 3 & 4 & 5 \\   \hline  f(x) & 4 & 1 & 3 & 5 & 2 \\  \hline \end{tabular}\]

If $u_0=4$ and $u_{n+1} = f(u_n)$ for $n \ge 0$, find $u_{2002}$

$\text{(A) }1 \qquad \text{(B) }2 \qquad \text{(C) }3 \qquad \text{(D) }4 \qquad \text{(E) }5$

Solution

Problem 10

Let $a$ and $b$ be distinct real numbers for which \[\frac{a}{b} + \frac{a+10b}{b+10a} = 2.\]

Find $\frac{a}{b}$

$\text{(A) }0.4 \qquad \text{(B) }0.5  \qquad \text{(C) }0.6 \qquad \text{(D) }0.7 \qquad \text{(E) }0.8$

Solution

Problem 11

Let $P(x)=kx^3 + 2k^2x^2+k^3.$ Find the sum of all real numbers $k$ for which $x-2$ is a factor of $P(x).$

$\text{(A) }-8 \qquad \text{(B) }-4 \qquad \text{(C) }0 \qquad \text{(D) }5 \qquad \text{(E) }8$

Solution

Problem 12

For $f_n(x)=x^n$ and $a \neq 1$ consider

$\text{I. } (f_{11}(a)f_{13}(a))^{14}$

$\text{II. } f_{11}(a)f_{13}(a)f_{14}(a)$

$\text{III. } (f_{11}(f_{13}(a)))^{14}$

$\text{IV. } f_{11}(f_{13}(f_{14}(a)))$

Which of these equal $f_{2002}(a)?$

$\text{(A) I and II only} \qquad \text{(B) II and III only} \qquad \text{(C) III and IV only} \qquad \text{(D) II, III, and IV only} \qquad \text{(E) all of them}$

Solution

Problem 13

Participation in the local soccer league this year is $10\%$ higher than last year. The number of males increased by $5\%$ and the number of females increased by $20\%$. What fraction of the soccer league is now female?

$\text{(A) }\frac{1}{3} \qquad \text{(B) }\frac{4}{11} \qquad \text{(C) }\frac{2}{5} \qquad \text{(D) }\frac{4}{9} \qquad \text{(E) }\frac{1}{2}$

Solution

Problem 14

The vertex $E$ of a square $EFGH$ is at the center of square $ABCD.$ The length of a side of $ABCD$ is $1$ and the length of a side of $EFGH$ is $2.$ Side $EF$ intersects $CD$ at $I$ and $EH$ intersects $AD$ at $J.$ If angle $EID=60^{\circ},$ the area of quadrilateral $EIDJ$ is

$\text{(A) }\frac{1}{4} \qquad \text{(B) }\frac{\sqrt{3}}{6} \qquad \text{(C) }\frac{1}{3} \qquad \text{(D) }\frac{\sqrt{2}}{4} \qquad \text{(E) }\frac{\sqrt{3}}{2}$

Solution

Problem 15

What is the smallest integer $n$ for which any subset of $\{ 1, 2, 3, \; \dots \; , 20 \}$ of size $n$ must contain two numbers that differ by 8?

$\text{(A) }2 \qquad \text{(B) }8 \qquad \text{(C) }12 \qquad \text{(D) }13 \qquad \text{(E) }15$

Solution

Problem 16

Two walls and the ceiling of a room meet at right angles at point $P.$ A fly is in the air one meter from one wall, eight meters from the other wall, and nine meters from point $P$. How many meters is the fly from the ceiling?

$\text{(A) }\sqrt{13} \qquad \text{(B) }\sqrt{14} \qquad \text{(C) }\sqrt{15} \qquad \text{(D) }4 \qquad \text{(E) }\sqrt{17}$

Solution

Problem 17

There are $1001$ red marbles and $1001$ black marbles in a box. Let $P_s$ be the probability that two marbles drawn at random from the box are the same color, and let $P_d$ be the probability that they are different colors. Find $|P_s-P_d|.$

$\text{(A) }0 \qquad \text{(B) }\frac{1}{2002} \qquad \text{(C) }\frac{1}{2001} \qquad \text{(D) }\frac {2}{2001} \qquad \text{(E) }\frac{1}{1000}$

Solution

Problem 18

For how many positive integers $n$ is $n^3 - 8n^2 + 20n - 13$ a prime number?

$\text{(A) one} \qquad \text{(B) two} \qquad \text{(C) three} \qquad \text{(D) four} \qquad \text{(E) more than four}$

Solution

Problem 19

If $a,b,c$ are real numbers such that $a^2 + 2b =7$, $b^2 + 4c= -7,$ and $c^2 + 6a= -14$, find $a^2 + b^2 + c^2.$

$\text{(A) }14 \qquad \text{(B) }21 \qquad \text{(C) }28 \qquad \text{(D) }35 \qquad \text{(E) }49$

Solution

Problem 20

How many three-digit numbers have at least one $2$ and at least one $3$?

$\text{(A) }52 \qquad \text{(B) }54  \qquad \text{(C) }56 \qquad \text{(D) }58 \qquad \text{(E) }60$

Solution

Problem 21

Let $f$ be a real-valued function such that

\[f(x) + 2f(\frac{2002}{x}) = 3x\]

for all $x>0.$ Find $f(2).$

$\text{(A) }1000 \qquad \text{(B) }2000 \qquad \text{(C) }3000 \qquad \text{(D) }4000 \qquad \text{(E) }6000$

Solution

Problem 22

In how many zeroes does the number $\frac{2002!}{(1001!)^2}$ end?

$\text{(A) }0 \qquad \text{(B) }1 \qquad \text{(C) }2 \qquad \text{(D) }200 \qquad \text{(E) }400$

Solution

Problem 23

Let \[a=\frac{1^2}{1} + \frac{2^2}{3} + \frac{3^2}{5} +  \; \dots \; + \frac{1001^2}{2001}\]

and

\[a=\frac{1^2}{3} + \frac{2^2}{5} + \frac{3^2}{7} +  \; \dots \; + \frac{1001^2}{2003}.\]

Find the integer closest to $a-b.$

$\text{(A) }500 \qquad \text{(B) }501 \qquad \text{(C) }999 \qquad \text{(D) }1000 \qquad \text{(E) }1001$

Solution

Problem 24

What is the maximum value of $n$ for which there is a set of distinct positive integers $k_1, k_2, ... k_n$ for which

\[k^2_1 + k^2_2 + ... + k^2_n = 2002?\]

$\text{(A) }14 \qquad \text{(B) }15 \qquad \text{(C) }16 \qquad \text{(D) }17 \qquad \text{(E) }18$

Solution

Problem 25

Under the new AMC $10, 12$ scoring method, $6$ points are given for each correct answer, $2.5$ points are given for each unanswered question, and no points are given for an incorrect answer. Some of the possible scores between $0$ and $150$ can be obtained in only one way, for example, the only way to obtain a score of $146.5$ is to have $24$ correct answers and one unanswered question. Some scores can be obtained in exactly two ways, for example, a score of $104.5$ can be obtained with $17$ correct answers, $1$ unanswered question, and $7$ incorrect, and also with $12$ correct answers and $13$ unanswered questions. There are (three) scores that can be obtained in exactly three ways. What is their sum?


$\text{(A) }175 \qquad \text{(B) }179.5 \qquad \text{(C) }182 \qquad \text{(D) }188.5 \qquad \text{(E) }201$

Solution

See also

2002 AMC 10P (ProblemsAnswer KeyResources)
Preceded by
2001 AMC 10 Problems
Followed by
2002 AMC 10A Problems
1 2 3 4 5 6 7 8 9 10 11 12 13 14 15 16 17 18 19 20 21 22 23 24 25
All AMC 10 Problems and Solutions

The problems on this page are copyrighted by the Mathematical Association of America's American Mathematics Competitions. AMC logo.png